How to solve this problem? I did not understand how to draw it's diagram. please help.












0












$begingroup$


At the midpoint of line segment $AB$ which is $p$ units long, a perpendicular $MR$ is erected with length $q$ units. An arc is described from $R$ with a radius equal to $frac{1}{2}AB$, meeting $AB$ at $T$. Then $AT$ and $TB$ are the roots of:



$textbf{(A)} x^2+px+q^2=0\ textbf{(B)} x^2-px+q^2=0\ textbf{(C)} x^2+px-q^2=0\ textbf{(D)} x^2-px-q^2=0\ textbf{(E)} x^2-px+q=0$










share|cite|improve this question











$endgroup$

















    0












    $begingroup$


    At the midpoint of line segment $AB$ which is $p$ units long, a perpendicular $MR$ is erected with length $q$ units. An arc is described from $R$ with a radius equal to $frac{1}{2}AB$, meeting $AB$ at $T$. Then $AT$ and $TB$ are the roots of:



    $textbf{(A)} x^2+px+q^2=0\ textbf{(B)} x^2-px+q^2=0\ textbf{(C)} x^2+px-q^2=0\ textbf{(D)} x^2-px-q^2=0\ textbf{(E)} x^2-px+q=0$










    share|cite|improve this question











    $endgroup$















      0












      0








      0





      $begingroup$


      At the midpoint of line segment $AB$ which is $p$ units long, a perpendicular $MR$ is erected with length $q$ units. An arc is described from $R$ with a radius equal to $frac{1}{2}AB$, meeting $AB$ at $T$. Then $AT$ and $TB$ are the roots of:



      $textbf{(A)} x^2+px+q^2=0\ textbf{(B)} x^2-px+q^2=0\ textbf{(C)} x^2+px-q^2=0\ textbf{(D)} x^2-px-q^2=0\ textbf{(E)} x^2-px+q=0$










      share|cite|improve this question











      $endgroup$




      At the midpoint of line segment $AB$ which is $p$ units long, a perpendicular $MR$ is erected with length $q$ units. An arc is described from $R$ with a radius equal to $frac{1}{2}AB$, meeting $AB$ at $T$. Then $AT$ and $TB$ are the roots of:



      $textbf{(A)} x^2+px+q^2=0\ textbf{(B)} x^2-px+q^2=0\ textbf{(C)} x^2+px-q^2=0\ textbf{(D)} x^2-px-q^2=0\ textbf{(E)} x^2-px+q=0$







      algebra-precalculus geometry






      share|cite|improve this question















      share|cite|improve this question













      share|cite|improve this question




      share|cite|improve this question








      edited Jan 23 at 5:43









      André 3000

      12.7k22243




      12.7k22243










      asked Jan 23 at 5:34









      AbhiAbhi

      11




      11






















          2 Answers
          2






          active

          oldest

          votes


















          0












          $begingroup$

          In such questions whenever you feel you are not able to visualize, and
          the problem involves both the concepts of geometry as well as
          algebraic equations always go with coordinate geometry or vectors.
          For the diagram you can consider points $A$ and $B$ as having the
          coordinates $(0,0)$ and $(p,0)$ since the line segment $AB$ is of
          length $p$ units.
          Now similarly calculate the midpoint coordinates $M$ then erect a
          perpendicular and find the coordinates of $R$. When you get the
          coordinates of $R$ just take it as a center and form the equation of
          circle using it.
          The equation of circle centered at a point $(h,k)$ is given by
          $$(x-h)^{2}+(y-k)^{2} = r^2$$
          Here the $(h,k)$ will be the coordinates of $R$ and the radius is
          given in the problem.
          For the final part you need to know the value of $T$. Now
          geometrically, when two shapes intersect like a circle and a line(in
          this case) there are intersection points. So just solve the equations
          of both circle and the line $AB$ ( which is nothing but the equation
          of $X$ axis). Simultaneously solving both the equations means that you
          want to find the intersection points of the circle and the line.
          Finally you will get the equation in terms of $x$,$p$ and $q$. Since
          it is a quadratic equation, there will be two roots which will
          correspond to $AT$ and $TB$.
          Hope this helps.......






          share|cite|improve this answer









          $endgroup$





















            0












            $begingroup$

            Construct the image step-bt-step,
            collecting and analyzing all the given information.




            1. Segment $AB$: $|AB|=p$.


            2. The point $M$ is the midpoint of $AB$,
              hence $|AM|=|MB|=tfrac p2$.


            3. From the point $M$ erect the perpendicular $MRperp AB$,
              of length $q$, that is, $|MR|=q$.



            Note that the radius $r$ of the arc from $R$
            is stated to be $tfrac12|AB|$, and the arc touches $AB$ at the point $T$.
            Than can only mean that
            the point $T$ is the midpoint of $AB$, hence $T=M$,
            $|AT|=|TB|=tfrac p2$ and also,
            $q=|MR|=r=tfrac12|AB|=tfrac p2$.



            Next, the algebraic part. We know that $|AT|$ and $|TB|$
            must be the roots of quadratic equation



            begin{align}
            (x-|AT|)(x-|BT|)&=
            x^2-(|AT|+|TB|)x+|AT|cdot|TB|=0
            tag{1}label{1}
            .
            end{align}



            Using all the previously collected pieces of information,
            namely that $|AT|+|TB|=p$, $|AT|=|TB|=tfrac12p=q$
            we can express eqref{1} as



            begin{align}
            x^2-px+q^2&=0
            tag{2}label{2}
            ,
            end{align}



            which has a match with one of the given options.






            share|cite|improve this answer











            $endgroup$













              Your Answer





              StackExchange.ifUsing("editor", function () {
              return StackExchange.using("mathjaxEditing", function () {
              StackExchange.MarkdownEditor.creationCallbacks.add(function (editor, postfix) {
              StackExchange.mathjaxEditing.prepareWmdForMathJax(editor, postfix, [["$", "$"], ["\\(","\\)"]]);
              });
              });
              }, "mathjax-editing");

              StackExchange.ready(function() {
              var channelOptions = {
              tags: "".split(" "),
              id: "69"
              };
              initTagRenderer("".split(" "), "".split(" "), channelOptions);

              StackExchange.using("externalEditor", function() {
              // Have to fire editor after snippets, if snippets enabled
              if (StackExchange.settings.snippets.snippetsEnabled) {
              StackExchange.using("snippets", function() {
              createEditor();
              });
              }
              else {
              createEditor();
              }
              });

              function createEditor() {
              StackExchange.prepareEditor({
              heartbeatType: 'answer',
              autoActivateHeartbeat: false,
              convertImagesToLinks: true,
              noModals: true,
              showLowRepImageUploadWarning: true,
              reputationToPostImages: 10,
              bindNavPrevention: true,
              postfix: "",
              imageUploader: {
              brandingHtml: "Powered by u003ca class="icon-imgur-white" href="https://imgur.com/"u003eu003c/au003e",
              contentPolicyHtml: "User contributions licensed under u003ca href="https://creativecommons.org/licenses/by-sa/3.0/"u003ecc by-sa 3.0 with attribution requiredu003c/au003e u003ca href="https://stackoverflow.com/legal/content-policy"u003e(content policy)u003c/au003e",
              allowUrls: true
              },
              noCode: true, onDemand: true,
              discardSelector: ".discard-answer"
              ,immediatelyShowMarkdownHelp:true
              });


              }
              });














              draft saved

              draft discarded


















              StackExchange.ready(
              function () {
              StackExchange.openid.initPostLogin('.new-post-login', 'https%3a%2f%2fmath.stackexchange.com%2fquestions%2f3084123%2fhow-to-solve-this-problem-i-did-not-understand-how-to-draw-its-diagram-please%23new-answer', 'question_page');
              }
              );

              Post as a guest















              Required, but never shown

























              2 Answers
              2






              active

              oldest

              votes








              2 Answers
              2






              active

              oldest

              votes









              active

              oldest

              votes






              active

              oldest

              votes









              0












              $begingroup$

              In such questions whenever you feel you are not able to visualize, and
              the problem involves both the concepts of geometry as well as
              algebraic equations always go with coordinate geometry or vectors.
              For the diagram you can consider points $A$ and $B$ as having the
              coordinates $(0,0)$ and $(p,0)$ since the line segment $AB$ is of
              length $p$ units.
              Now similarly calculate the midpoint coordinates $M$ then erect a
              perpendicular and find the coordinates of $R$. When you get the
              coordinates of $R$ just take it as a center and form the equation of
              circle using it.
              The equation of circle centered at a point $(h,k)$ is given by
              $$(x-h)^{2}+(y-k)^{2} = r^2$$
              Here the $(h,k)$ will be the coordinates of $R$ and the radius is
              given in the problem.
              For the final part you need to know the value of $T$. Now
              geometrically, when two shapes intersect like a circle and a line(in
              this case) there are intersection points. So just solve the equations
              of both circle and the line $AB$ ( which is nothing but the equation
              of $X$ axis). Simultaneously solving both the equations means that you
              want to find the intersection points of the circle and the line.
              Finally you will get the equation in terms of $x$,$p$ and $q$. Since
              it is a quadratic equation, there will be two roots which will
              correspond to $AT$ and $TB$.
              Hope this helps.......






              share|cite|improve this answer









              $endgroup$


















                0












                $begingroup$

                In such questions whenever you feel you are not able to visualize, and
                the problem involves both the concepts of geometry as well as
                algebraic equations always go with coordinate geometry or vectors.
                For the diagram you can consider points $A$ and $B$ as having the
                coordinates $(0,0)$ and $(p,0)$ since the line segment $AB$ is of
                length $p$ units.
                Now similarly calculate the midpoint coordinates $M$ then erect a
                perpendicular and find the coordinates of $R$. When you get the
                coordinates of $R$ just take it as a center and form the equation of
                circle using it.
                The equation of circle centered at a point $(h,k)$ is given by
                $$(x-h)^{2}+(y-k)^{2} = r^2$$
                Here the $(h,k)$ will be the coordinates of $R$ and the radius is
                given in the problem.
                For the final part you need to know the value of $T$. Now
                geometrically, when two shapes intersect like a circle and a line(in
                this case) there are intersection points. So just solve the equations
                of both circle and the line $AB$ ( which is nothing but the equation
                of $X$ axis). Simultaneously solving both the equations means that you
                want to find the intersection points of the circle and the line.
                Finally you will get the equation in terms of $x$,$p$ and $q$. Since
                it is a quadratic equation, there will be two roots which will
                correspond to $AT$ and $TB$.
                Hope this helps.......






                share|cite|improve this answer









                $endgroup$
















                  0












                  0








                  0





                  $begingroup$

                  In such questions whenever you feel you are not able to visualize, and
                  the problem involves both the concepts of geometry as well as
                  algebraic equations always go with coordinate geometry or vectors.
                  For the diagram you can consider points $A$ and $B$ as having the
                  coordinates $(0,0)$ and $(p,0)$ since the line segment $AB$ is of
                  length $p$ units.
                  Now similarly calculate the midpoint coordinates $M$ then erect a
                  perpendicular and find the coordinates of $R$. When you get the
                  coordinates of $R$ just take it as a center and form the equation of
                  circle using it.
                  The equation of circle centered at a point $(h,k)$ is given by
                  $$(x-h)^{2}+(y-k)^{2} = r^2$$
                  Here the $(h,k)$ will be the coordinates of $R$ and the radius is
                  given in the problem.
                  For the final part you need to know the value of $T$. Now
                  geometrically, when two shapes intersect like a circle and a line(in
                  this case) there are intersection points. So just solve the equations
                  of both circle and the line $AB$ ( which is nothing but the equation
                  of $X$ axis). Simultaneously solving both the equations means that you
                  want to find the intersection points of the circle and the line.
                  Finally you will get the equation in terms of $x$,$p$ and $q$. Since
                  it is a quadratic equation, there will be two roots which will
                  correspond to $AT$ and $TB$.
                  Hope this helps.......






                  share|cite|improve this answer









                  $endgroup$



                  In such questions whenever you feel you are not able to visualize, and
                  the problem involves both the concepts of geometry as well as
                  algebraic equations always go with coordinate geometry or vectors.
                  For the diagram you can consider points $A$ and $B$ as having the
                  coordinates $(0,0)$ and $(p,0)$ since the line segment $AB$ is of
                  length $p$ units.
                  Now similarly calculate the midpoint coordinates $M$ then erect a
                  perpendicular and find the coordinates of $R$. When you get the
                  coordinates of $R$ just take it as a center and form the equation of
                  circle using it.
                  The equation of circle centered at a point $(h,k)$ is given by
                  $$(x-h)^{2}+(y-k)^{2} = r^2$$
                  Here the $(h,k)$ will be the coordinates of $R$ and the radius is
                  given in the problem.
                  For the final part you need to know the value of $T$. Now
                  geometrically, when two shapes intersect like a circle and a line(in
                  this case) there are intersection points. So just solve the equations
                  of both circle and the line $AB$ ( which is nothing but the equation
                  of $X$ axis). Simultaneously solving both the equations means that you
                  want to find the intersection points of the circle and the line.
                  Finally you will get the equation in terms of $x$,$p$ and $q$. Since
                  it is a quadratic equation, there will be two roots which will
                  correspond to $AT$ and $TB$.
                  Hope this helps.......







                  share|cite|improve this answer












                  share|cite|improve this answer



                  share|cite|improve this answer










                  answered Jan 23 at 6:08









                  SNEHIL SANYALSNEHIL SANYAL

                  616110




                  616110























                      0












                      $begingroup$

                      Construct the image step-bt-step,
                      collecting and analyzing all the given information.




                      1. Segment $AB$: $|AB|=p$.


                      2. The point $M$ is the midpoint of $AB$,
                        hence $|AM|=|MB|=tfrac p2$.


                      3. From the point $M$ erect the perpendicular $MRperp AB$,
                        of length $q$, that is, $|MR|=q$.



                      Note that the radius $r$ of the arc from $R$
                      is stated to be $tfrac12|AB|$, and the arc touches $AB$ at the point $T$.
                      Than can only mean that
                      the point $T$ is the midpoint of $AB$, hence $T=M$,
                      $|AT|=|TB|=tfrac p2$ and also,
                      $q=|MR|=r=tfrac12|AB|=tfrac p2$.



                      Next, the algebraic part. We know that $|AT|$ and $|TB|$
                      must be the roots of quadratic equation



                      begin{align}
                      (x-|AT|)(x-|BT|)&=
                      x^2-(|AT|+|TB|)x+|AT|cdot|TB|=0
                      tag{1}label{1}
                      .
                      end{align}



                      Using all the previously collected pieces of information,
                      namely that $|AT|+|TB|=p$, $|AT|=|TB|=tfrac12p=q$
                      we can express eqref{1} as



                      begin{align}
                      x^2-px+q^2&=0
                      tag{2}label{2}
                      ,
                      end{align}



                      which has a match with one of the given options.






                      share|cite|improve this answer











                      $endgroup$


















                        0












                        $begingroup$

                        Construct the image step-bt-step,
                        collecting and analyzing all the given information.




                        1. Segment $AB$: $|AB|=p$.


                        2. The point $M$ is the midpoint of $AB$,
                          hence $|AM|=|MB|=tfrac p2$.


                        3. From the point $M$ erect the perpendicular $MRperp AB$,
                          of length $q$, that is, $|MR|=q$.



                        Note that the radius $r$ of the arc from $R$
                        is stated to be $tfrac12|AB|$, and the arc touches $AB$ at the point $T$.
                        Than can only mean that
                        the point $T$ is the midpoint of $AB$, hence $T=M$,
                        $|AT|=|TB|=tfrac p2$ and also,
                        $q=|MR|=r=tfrac12|AB|=tfrac p2$.



                        Next, the algebraic part. We know that $|AT|$ and $|TB|$
                        must be the roots of quadratic equation



                        begin{align}
                        (x-|AT|)(x-|BT|)&=
                        x^2-(|AT|+|TB|)x+|AT|cdot|TB|=0
                        tag{1}label{1}
                        .
                        end{align}



                        Using all the previously collected pieces of information,
                        namely that $|AT|+|TB|=p$, $|AT|=|TB|=tfrac12p=q$
                        we can express eqref{1} as



                        begin{align}
                        x^2-px+q^2&=0
                        tag{2}label{2}
                        ,
                        end{align}



                        which has a match with one of the given options.






                        share|cite|improve this answer











                        $endgroup$
















                          0












                          0








                          0





                          $begingroup$

                          Construct the image step-bt-step,
                          collecting and analyzing all the given information.




                          1. Segment $AB$: $|AB|=p$.


                          2. The point $M$ is the midpoint of $AB$,
                            hence $|AM|=|MB|=tfrac p2$.


                          3. From the point $M$ erect the perpendicular $MRperp AB$,
                            of length $q$, that is, $|MR|=q$.



                          Note that the radius $r$ of the arc from $R$
                          is stated to be $tfrac12|AB|$, and the arc touches $AB$ at the point $T$.
                          Than can only mean that
                          the point $T$ is the midpoint of $AB$, hence $T=M$,
                          $|AT|=|TB|=tfrac p2$ and also,
                          $q=|MR|=r=tfrac12|AB|=tfrac p2$.



                          Next, the algebraic part. We know that $|AT|$ and $|TB|$
                          must be the roots of quadratic equation



                          begin{align}
                          (x-|AT|)(x-|BT|)&=
                          x^2-(|AT|+|TB|)x+|AT|cdot|TB|=0
                          tag{1}label{1}
                          .
                          end{align}



                          Using all the previously collected pieces of information,
                          namely that $|AT|+|TB|=p$, $|AT|=|TB|=tfrac12p=q$
                          we can express eqref{1} as



                          begin{align}
                          x^2-px+q^2&=0
                          tag{2}label{2}
                          ,
                          end{align}



                          which has a match with one of the given options.






                          share|cite|improve this answer











                          $endgroup$



                          Construct the image step-bt-step,
                          collecting and analyzing all the given information.




                          1. Segment $AB$: $|AB|=p$.


                          2. The point $M$ is the midpoint of $AB$,
                            hence $|AM|=|MB|=tfrac p2$.


                          3. From the point $M$ erect the perpendicular $MRperp AB$,
                            of length $q$, that is, $|MR|=q$.



                          Note that the radius $r$ of the arc from $R$
                          is stated to be $tfrac12|AB|$, and the arc touches $AB$ at the point $T$.
                          Than can only mean that
                          the point $T$ is the midpoint of $AB$, hence $T=M$,
                          $|AT|=|TB|=tfrac p2$ and also,
                          $q=|MR|=r=tfrac12|AB|=tfrac p2$.



                          Next, the algebraic part. We know that $|AT|$ and $|TB|$
                          must be the roots of quadratic equation



                          begin{align}
                          (x-|AT|)(x-|BT|)&=
                          x^2-(|AT|+|TB|)x+|AT|cdot|TB|=0
                          tag{1}label{1}
                          .
                          end{align}



                          Using all the previously collected pieces of information,
                          namely that $|AT|+|TB|=p$, $|AT|=|TB|=tfrac12p=q$
                          we can express eqref{1} as



                          begin{align}
                          x^2-px+q^2&=0
                          tag{2}label{2}
                          ,
                          end{align}



                          which has a match with one of the given options.







                          share|cite|improve this answer














                          share|cite|improve this answer



                          share|cite|improve this answer








                          edited Jan 23 at 7:36

























                          answered Jan 23 at 7:25









                          g.kovg.kov

                          6,3071818




                          6,3071818






























                              draft saved

                              draft discarded




















































                              Thanks for contributing an answer to Mathematics Stack Exchange!


                              • Please be sure to answer the question. Provide details and share your research!

                              But avoid



                              • Asking for help, clarification, or responding to other answers.

                              • Making statements based on opinion; back them up with references or personal experience.


                              Use MathJax to format equations. MathJax reference.


                              To learn more, see our tips on writing great answers.




                              draft saved


                              draft discarded














                              StackExchange.ready(
                              function () {
                              StackExchange.openid.initPostLogin('.new-post-login', 'https%3a%2f%2fmath.stackexchange.com%2fquestions%2f3084123%2fhow-to-solve-this-problem-i-did-not-understand-how-to-draw-its-diagram-please%23new-answer', 'question_page');
                              }
                              );

                              Post as a guest















                              Required, but never shown





















































                              Required, but never shown














                              Required, but never shown












                              Required, but never shown







                              Required, but never shown

































                              Required, but never shown














                              Required, but never shown












                              Required, but never shown







                              Required, but never shown







                              Popular posts from this blog

                              Can a sorcerer learn a 5th-level spell early by creating spell slots using the Font of Magic feature?

                              Does disintegrating a polymorphed enemy still kill it after the 2018 errata?

                              A Topological Invariant for $pi_3(U(n))$